Questions about the world of GMAT Math from other sources and general math related questions.
Luci
 
 

4GMAT-quiz

by Luci Sun Jul 29, 2007 6:35 pm

What will be the remainder when 13^7 + 14^7 + 15^7 + 16^7 is divided by 58?
57
1
30
0
28


The answer is D) 0, but how do you reach the solution?
Luci
 
 

by Luci Sun Jul 29, 2007 6:42 pm

Actually I´m realicing it right now that 13+14+15+16=58

I guess if you power each of these numbers to the same power (7), division by the same number (58) will yield the same result? 0 in this case.

Can anyone confirm this please?

Thanks
Harish Dorai
 
 

by Harish Dorai Mon Jul 30, 2007 10:26 am

I am not sure about the rule that you mentioned. This is how I approached and I narrowed it down to 3 choices now (C), (D) and (E). This is what I did (it is a little bit lengthy). Even though it looks lengthy and complicated on paper, it doesn't take much time to think through this.

Step 1: Find the units digit of 13^7 + 14^7 + 15^7 + 16^7.

For 13^7 the units digit is 7, for 14^7 the units digit is 4, for 15^7 the units digit is 5 and for 16^7 it is 6.

7 + 4 + 5 + 6 = 22, which means the units digit of 13^7 + 14^7 + 15^7 + 16^7 is 2.


Step 2:

Let us assume the the above sum is some big number, XYZ.......2 (the last digit is 2 as we figured out in Step 2). And we will see what could be the remainders by playing with the answer choices.

If you examine answer choice (A), it says the reminder is 57. In order for us to get a remainder 57, for a number with units digit 2 divided by 58, we should have a multiple that ends in 5 for the number 58. I will explain it using some small numbers.

If you divide 352 by 59, we will get a remainder 57. The calculation is depicted below.

5
------------
59) 3 5 2
2 9 5
--------
5 7

The very reason why we got a 57 above (or a remainder with units digit 7) is because we have a multiple for 59 that ends in 5 which is 295 which when subtracted from 352 yields the remainder 57.

Now coming back to our question, there is no multiple for 58 that ends in 5. So 57 cannot be a remainder. So we can eliminate answer choice (A).

Similarly we can eliminate answer choice (B) also. In order for us to get a remainder 1, we should have a multiple for 58 that ends in 1. We know there is no multiple for 58 that ends in 1.

Like above we cannot really eliminate the remainders 28, 30 and 0. So I am stuck here.
StaceyKoprince
ManhattanGMAT Staff
 
Posts: 9355
Joined: Wed Oct 19, 2005 9:05 am
Location: Montreal
 

by StaceyKoprince Tue Jul 31, 2007 1:08 am

Can you elaborate on the source of this question? I don't recognize "4GMAT"
thanks!
Stacey Koprince
Instructor
Director, Content & Curriculum
ManhattanPrep
Luci
 
 

by Luci Tue Jul 31, 2007 12:19 pm

Not much, this is the website: http://www.4gmat.com/ and there they have a Free GMAT math test with 10 questions.
After completing the test they tell you the answers but dont explain them.
I had doubts with that particular answer

Thanks
Luci
 
 

by Luci Thu Aug 02, 2007 6:04 pm

No responses for this question?

Thanks
StaceyKoprince
ManhattanGMAT Staff
 
Posts: 9355
Joined: Wed Oct 19, 2005 9:05 am
Location: Montreal
 

by StaceyKoprince Tue Aug 07, 2007 1:47 pm

This is an... odd... one. I've never seen the test require us to know this particular theorem. I'm not sure I'd use this source to study.

58 = 2*29
I need to figure out the remainder when the sum is divisible by 2 and also by 29.

13^7 + 14^7 + 15^7 + 16^7
odd + even + odd + even = even, so this is divisible by 2 with a remainder of zero.

This part will be harder to follow. When evaluating remainders when dividing by 29, 15 can also be written as -14 and 16 can also be written as -13 (because 29 + (-14) = 15 and 29 + (-13) = 16). This lets us re-write the expression:
13^7 + 14^7 + (-14)^7 + (-13)^7
The middle two terms cancel and the outer two terms cancel, leaving a sum of zero. Zero divided by 29 gives a remainder of zero.

Overall, then, I have a remainder of zero. Again, I have NEVER (in more than ten years) seen a test question that required this knowledge so I really wouldn't worry about this.
Stacey Koprince
Instructor
Director, Content & Curriculum
ManhattanPrep
shaji
 
 

by shaji Wed Aug 08, 2007 8:39 am

The concept here is the 'Remainder Theorem' and this problem is a 15seconds job.

Well!!! good the GMAT hasn't got into the habbit of 'surprises'. If they do take a fancy to such stuff ,perhaps, the tests would be more 'challenging'


skoprince Wrote:This is an... odd... one. I've never seen the test require us to know this particular theorem. I'm not sure I'd use this source to study.

58 = 2*29
I need to figure out the remainder when the sum is divisible by 2 and also by 29.

13^7 + 14^7 + 15^7 + 16^7
odd + even + odd + even = even, so this is divisible by 2 with a remainder of zero.

This part will be harder to follow. When evaluating remainders when dividing by 29, 15 can also be written as -14 and 16 can also be written as -13 (because 29 + (-14) = 15 and 29 + (-13) = 16). This lets us re-write the expression:
13^7 + 14^7 + (-14)^7 + (-13)^7
The middle two terms cancel and the outer two terms cancel, leaving a sum of zero. Zero divided by 29 gives a remainder of zero.

Overall, then, I have a remainder of zero. Again, I have NEVER (in more than ten years) seen a test question that required this knowledge so I really wouldn't worry about this.
Guest
 
 

by Guest Wed Aug 08, 2007 10:54 am

Shaji, could you please explain the 15 seconds approach here by Remainder theorem here?

Thanks
JAMGAJR
 
 

Remainder

by JAMGAJR Wed Aug 08, 2007 11:33 am

Another way

(15^7 + 14^7) + (13^7 + 16^7).......... remember that a^n + b^n is always divisible by a+b when n is odd

so both of the expressions are divisible by 29

so 1/2* ( odd + even + odd + even)= 1/2*(even)= divisible


remainder 0
shaji
 
 

by shaji Thu Aug 09, 2007 11:47 am

JAMGAJR has it for you; the corolory of the remainder theorem.

Anonymous Wrote:Shaji, could you please explain the 15 seconds approach here by Remainder theorem here?

Thanks
StaceyKoprince
ManhattanGMAT Staff
 
Posts: 9355
Joined: Wed Oct 19, 2005 9:05 am
Location: Montreal
 

by StaceyKoprince Sun Aug 12, 2007 12:05 am

Well!!! good the GMAT hasn't got into the habbit of 'surprises'. If they do take a fancy to such stuff ,perhaps, the tests would be more 'challenging'


Just note that this question didn't come from GMATPrep or OG - it came from some other source - so this has no bearing on whether the official GMAT may or will use such concepts on the real test.
Stacey Koprince
Instructor
Director, Content & Curriculum
ManhattanPrep
anadi
 
 

here is a solution

by anadi Mon Aug 13, 2007 2:15 am

That might make it look more like a gmat question and concept

(a-b)^2 = a^2+b^2-2ab
(a-b)^3 = a^3-b^3+3ab^2-3ba^2
If we expand any power of a+b like this, there will be one and only one part of multinomial that will have either a or b in their highest power.
now,
13^7+14^7+15^7+16^7 can be written as,
13^7+14^7+(29-14)^7+(29-13)^7, when we expand the later 2 parts, (-14)^7 and (-13)^7 wil be the only once that will not have 29(or a power of 29) multiplied to them. So 13^7+14^7+(-14)^7+(-13)^7 = 0.

Rest of the expanded part will have a 29(or a power of 29) as a factor in each, and from the original 13^7+14^7+15^7+16^7, we know that this must be even. Hence we got 2 and 29 as factors of equation and that proves this as well as the remainder theorem.
This applies to only odd powers since even powers of -14 and -29 would not have cancelled out with the positive ones.